Question

In a Portfolio Selection problem, let X1, X2 and 3 represent the number of shares purchased for stocks 1, 2 and 3, which have
0 0
Add a comment Improve this question Transcribed image text
Answer #1

Hello

Q - 1 - OPTION A is correct

Given the prices and share names and the maximum amount of $40000 to invest,

The appropriate constraint would be:

$45x_1+\$15x_2+\$100x_3 \leq \$40,000

Q - 2 - OPTION B is correct.

\\ x_1\leq 0.35*(x_1+x_2+x_3 ) \\ \\ 0.65x_1-0.35x_2-0.35x_3\leq 0

Thanks!

Please drop an upvote if you find this helpful.

Add a comment
Know the answer?
Add Answer to:
In a Portfolio Selection problem, let X1, X2 and 3 represent the number of shares purchased...
Your Answer:

Post as a guest

Your Name:

What's your source?

Earn Coins

Coins can be redeemed for fabulous gifts.

Not the answer you're looking for? Ask your own homework help question. Our experts will answer your question WITHIN MINUTES for Free.
Similar Homework Help Questions
ADVERTISEMENT
Free Homework Help App
Download From Google Play
Scan Your Homework
to Get Instant Free Answers
Need Online Homework Help?
Ask a Question
Get Answers For Free
Most questions answered within 3 hours.
ADVERTISEMENT
ADVERTISEMENT
ADVERTISEMENT